At one time, many astronomers assumed that Earth remains motionless while the stars revolve around it. They concluded...

Anna20 on July 21, 2020

Assumption

Why is B incorrect here? Please can you set out the explanation of the argument and the correct answer? Thank you!!

Reply
Create a free account to read and take part in forum discussions.

Already have an account? log in

shunhe on July 22, 2020

Hi @Anna2020,

Thanks for the question! So let’s take a look at the stimulus. We’re told that astronomers used to think that the Earth remains motionless while stars revolve around it, and concluded based on this fact that stars can’t have been too far from Earth. Why? Well, if the stars are super far away, they have to be move super quickly in order to circle Earth during the day and get back to roughly the same places during the night.

So now we’re asked for an assumption required by the reasoning above; in other words, this is a strengthen with necessary premise question. And we can realize that the astronomers went from thinking that if the stars are far away then they have to move super fast to thinking that the stars aren’t far away. That means they have to believe that the stars don’t move super fast! And that’s just the following logic:

Premise 1: Stars far —> Move super fast ?Assumption: ~Move super fast
Conclusion: ~Stars far

And this is what (D) tells us, that the argument assumes that stars don’t move at tremendously great speeds. Let’s try out the negation test; what if stars do move at tremendously great speeds? Then the astronomers can’t get to their conclusion; it breaks the argument, and so (D) is a necessary assumption.

(B), on the other hand, tells us that stars move at the exact same speed when revolving around the Earth. Is this a necessary assumption? No, not at all, and we can use the negation test to see why. Let’s say that the stars don’t move at the exact same speed. Some move at three million miles per second, while there’s one that moves at 2.99 million miles per second. Does that break the argument? No, not at all. It’s perfectly consistent with the astronomers’ reasoning, and so (B) is wrong.

Hope this helps! Feel free to ask any other questions that you might have.